Difference between revisions of "2025 AIME I Problems/Problem 14"
(Created page with "This problem does not exist yet") |
m (→Solution 1: fixed incorrect logic) |
||
(8 intermediate revisions by 4 users not shown) | |||
Line 1: | Line 1: | ||
− | + | ==Problem== | |
+ | Let <math>ABCDE</math> be a convex pentagon with <math>AB=14,</math> <math>BC=7,</math> <math>CD=24,</math> <math>DE=13,</math> <math>EA=26,</math> and <math>\angle B=\angle E=60^{\circ}.</math> For each point <math>X</math> in the plane, define <math>f(X)=AX+BX+CX+DX+EX.</math> The least possible value of <math>f(X)</math> can be expressed as <math>m+n\sqrt{p},</math> where <math>m</math> and <math>n</math> are positive integers and <math>p</math> is not divisible by the square of any prime. Find <math>m+n+p.</math> | ||
+ | ==Solution 1== | ||
+ | Assume <math>AX=a, BX=b, CX=c</math>, by Ptolemy inequality we have <math>a+2b\geq \sqrt{3}XE; a+2c\geq \sqrt{3}BX</math>, while the inequality is reached when both <math>CXAB</math> and <math>AXDE</math> are concyclic. Since <math>\angle{BXA}=\angle{BCA}=\angle{EDA}=\angle{EXA}=90^{\circ}</math>, so <math>B,X,E</math> lie on the same line. Thus, the desired value is then <math>(1+\frac{\sqrt{3}}{2})BE</math>. | ||
+ | Note <math>\cos(\angle{DAC})=\frac{1}{7}, \cos (\angle{EAB})=-\frac{11}{14}, BE=38</math> by LOC, the answer is then <math>38+19\sqrt{3}\implies \boxed{060}</math> | ||
+ | ~ Bluesoul | ||
+ | ==Solution 2== | ||
+ | <asy> | ||
+ | size(10cm); | ||
+ | import math; import geometry; import olympiad; | ||
+ | point A,B,C,D,F,P,X; A=(0,-7sqrt(3)); B=(-7,0); C=(0,0); D=(156/7,-36sqrt(3)/7); F=(169/7,-88sqrt(3)/7); P=(132/7,60sqrt(3)/7); X=(8580/2527,-10604sqrt(3)/2527); | ||
+ | draw(A--B--C--P--D--F--A--C--D--A--P); draw(B--F); draw(circumcircle(A,B,C)); draw(circumcircle(A,D,F)); draw(circumcircle(C,P,D)); draw(C--X--D); | ||
+ | label("A",A,SE); label("B",B,W); label("C",C,NW); label("P",P,N); label("D",D,E); label("E",F,SE); label("X",X,E); | ||
+ | </asy> | ||
+ | Firstly, note that <math>\triangle ABC</math> and <math>\triangle ADE</math> are just 30-60-90 triangles. Let <math>X</math> be the Fermat point of <math>\triangle ACD</math>, with motivation stemming from considering the pentagon as <math>\triangle ACD</math> with the two 30-60-90 extensions. Note that <math>AX+CX+DX</math> is minimized at this point when <math>\angle AXC=\angle CXD=\angle AXD=120^{\circ}</math>. Because we have <math>\angle ABC=\angle AED=60^{\circ}</math>, then <math>ABCX</math> and <math>AXDE</math> are both cyclic. Then we have <math>\angle AXE=\angle ADE=90^{\circ}</math> and <math>\angle BXA=\angle BCA=90^{\circ}</math>. Then it turns out that we actually have <math>\angle BXE=90^{\circ}+90^{\circ}=180^{\circ}</math>, implying that <math>B</math>, <math>X</math> and <math>E</math> are collinear. Now, by the triangle inequality, we must have <math>BX+XE\geq BE</math>, with equality occurring when <math>X</math> is on <math>BE</math>. Thus <math>AX+CX+DX</math> and <math>BX+EX</math> are minimized, so this point <math>X</math> is our desired point. | ||
− | + | Firstly, we will find <math>BX+EX=BE</math>. We have that <math>AC=7\sqrt{3}</math> and <math>AD=13\sqrt{3}</math>, so applying the Law of Cosines in <math>\triangle ACD</math>, we get <cmath>147+507-2(7\sqrt{3})(13\sqrt{3})\cos (\angle CAD)=576\implies \cos(\angle CAD)=\frac{1}{7}.</cmath> It follows as a result that <math>\sin (\angle CAD)=\frac{4\sqrt{3}}{7}</math>. Then we want to find <math>\cos (\angle BAE)</math>. We can do this by seeing <cmath>\cos (\angle BAE)=\cos (\angle CAD+60^{\circ})=\cos (\angle CAD)\cos 60^{\circ}-\sin (\angle CAD)\sin 60^{\circ}=\frac{1}{7}\cdot \frac{1}{2}-\frac{4\sqrt{3}}{7}\cdot \frac{\sqrt{3}}{2}=-\frac{11}{14}.</cmath> Applying the Law of Cosines again in <math>\triangle BAE</math>, then because <math>AB=14</math> and <math>AE=26</math>, we have <cmath>14^2+26^2-2(14)(26)\left (-\frac{11}{14}\right )=196+676-2\cdot 26\cdot (-11)=872+572=1444=BE^2,</cmath> so it follows that <math>BE=38=BX+EX</math>. | |
+ | |||
+ | Now, we will find the value of <math>AX+CX+DX</math>. Construct a point <math>P</math> outside such that <math>\triangle CPD</math> is equilateral, as shown. By property of fermat point, then <math>A</math>, <math>X</math>, and <math>P</math> are collinear. Additionally, <math>\angle CXD=120^{\circ}</math>, so <math>CPDX</math> is cyclic. Applying Ptolemy's Theorem, we have that <math>(CX)(PD)+(CP)(XD)=(XP)(CD)</math>. But since <math>\triangle CPD</math> is equilateral, it follows that <math>CX+DX=PX</math>. Then <math>AX+CX+DX=AX+PX=AP</math>, so we wish to find <math>AP</math>. Applying the Law of Cosines in <math>\triangle ACD</math>, we have that <cmath>(13\sqrt{3})^2+24^2-2(13\sqrt{3})(24)\cos (\angle ADC)=(7\sqrt{3})^2\implies \cos (\angle ADC)=\frac{\sqrt{3}}{2}\implies \angle ADC=30^{\circ}.</cmath> Then because <math>\angle CDP=60^{\circ}</math>, then <math>\angle ADP=90^{\circ}</math>, so we can find <math>AP</math> simply with the Pythagorean Theorem. We know <math>AD=13\sqrt{3}</math> and <math>DP=CD=24</math>, so <math>AP=\sqrt{(13\sqrt{3})^2+24^2}=19\sqrt{3}</math>. | ||
+ | |||
+ | We then have <math>f(X)=AX+BX+CX+DX+EX=(BX+EX)+(AX+CX+DX)=BE+AP=38+19\sqrt{3}</math>, which is our minimum value. Therefore, the answer to the problem is <math>38+19+3=\boxed{060}</math>. | ||
+ | |||
+ | ~ethanzhang1001 | ||
+ | |||
+ | ==See also== | ||
+ | {{AIME box|year=2025|num-b=13|num-a=15|n=I}} | ||
+ | |||
+ | {{MAA Notice}} |
Latest revision as of 12:36, 14 February 2025
Contents
Problem
Let be a convex pentagon with
and
For each point
in the plane, define
The least possible value of
can be expressed as
where
and
are positive integers and
is not divisible by the square of any prime. Find
Solution 1
Assume , by Ptolemy inequality we have
, while the inequality is reached when both
and
are concyclic. Since
, so
lie on the same line. Thus, the desired value is then
.
Note by LOC, the answer is then
~ Bluesoul
Solution 2
Firstly, note that
and
are just 30-60-90 triangles. Let
be the Fermat point of
, with motivation stemming from considering the pentagon as
with the two 30-60-90 extensions. Note that
is minimized at this point when
. Because we have
, then
and
are both cyclic. Then we have
and
. Then it turns out that we actually have
, implying that
,
and
are collinear. Now, by the triangle inequality, we must have
, with equality occurring when
is on
. Thus
and
are minimized, so this point
is our desired point.
Firstly, we will find . We have that
and
, so applying the Law of Cosines in
, we get
It follows as a result that
. Then we want to find
. We can do this by seeing
Applying the Law of Cosines again in
, then because
and
, we have
so it follows that
.
Now, we will find the value of . Construct a point
outside such that
is equilateral, as shown. By property of fermat point, then
,
, and
are collinear. Additionally,
, so
is cyclic. Applying Ptolemy's Theorem, we have that
. But since
is equilateral, it follows that
. Then
, so we wish to find
. Applying the Law of Cosines in
, we have that
Then because
, then
, so we can find
simply with the Pythagorean Theorem. We know
and
, so
.
We then have , which is our minimum value. Therefore, the answer to the problem is
.
~ethanzhang1001
See also
2025 AIME I (Problems • Answer Key • Resources) | ||
Preceded by Problem 13 |
Followed by Problem 15 | |
1 • 2 • 3 • 4 • 5 • 6 • 7 • 8 • 9 • 10 • 11 • 12 • 13 • 14 • 15 | ||
All AIME Problems and Solutions |
The problems on this page are copyrighted by the Mathematical Association of America's American Mathematics Competitions.